Density matrix, change of basis, I don't understand the basics

In summary: Te_n\rangle##. So, to get the matrix of the density operator in the ##\left|z_+ \right>,\left|z_- \right>## basis, you would need to solve for ##T_{ij}## in terms of ##e_i,e_j,\ldots,e_n##.
  • #36
I haven't actually solved this problem. All I said was this:

Fredrik said:
I haven't worked out the solution to this problem, so I don't know what exactly will be useful. But the first thing that comes to mind is that every vector in this space is an eigenstate of an operator of the form ##\vec n\cdot\vec S##, where ##\vec n## is some unit vector in ##\mathbb R^3##. This seems relevant, since you're asked to use a rotation operator.
I'll give it a shot and see if I can think of something.
 
Physics news on Phys.org
  • #37
I've edited my last post, I actually found the modulus squared of alpha in terms of theta, but I am not sure I didn't make any algebra mistake.
 
Last edited:
  • #38
Working with the modulus squared will lose you any phase information. Really the point I was trying to make earlier about the equality ##|\alpha|^2+|\beta|^2=1## was I was hoping you would realize that it looks awfully similar to the trigonometric identity ##\sin^2(\theta)+\cos^2(\theta)=1##.
 
  • #39
Matterwave said:
Working with the modulus squared will lose you any phase information. Really the point I was trying to make earlier about the equality ##|\alpha|^2+|\beta|^2=1## was I was hoping you would realize that it looks awfully similar to the trigonometric identity ##\sin^2(\theta)+\cos^2(\theta)=1##.

I see but that's basically what I reached then.
Because ##|\alpha | ^2=\frac{1+ \cos \theta }{2}=\cos ^2 \left ( \frac{\theta}{2} \right )## and ##|\beta |^2 =\frac{1-\cos \theta}{2} = \sin ^2 \left ( \frac{\theta}{2} \right )##.
 
  • #40
Yes, but that form is not conducive to writing out ##\alpha,\beta##.
 
  • #41
Matterwave said:
Yes, but that form is not conducive to writing out ##\alpha,\beta##.

hmm right I lose the phase information as you said.
Edit: I'll try to see if I can manage to get alpha and beta in terms of the angles, knowing their modulus squared.
 
  • #42
Make sure you keep track of the different angles that appear in this calculation. For one, the ##\theta## I defined in order to parametrize ##\alpha,\beta## will not necessarily have anything to do with the (2 other ) ##\theta##'s previously defined.

The point I was trying to get at is since you noticed that ##\sin^2(\theta)+\cos^2(\theta)=1## shouldn't I be able to write my ##\alpha,\beta## as ##\alpha=\sin(\theta')e^{i\phi'}## and ##\beta=\cos(\theta')e^{i\eta'}##? If I put this in my standard special unitary matrix form, how is this special unitary matrix decomposed into ##U=\cos(\theta/2)\hat{I}+\sin(\theta/2)\hat{n}\cdot\vec{S}##?
 
  • #43
I found a solution, sort of. But maybe you guys are already past this stage. I haven't been following the thread closely.

Let a,b be the unique complex numbers such that ##|\chi\rangle=a|+\rangle+b|-\rangle##. We have ##\rho=|\chi\rangle\langle\chi|##. Let ##|\phi\rangle## be any unit vector that's orthogonal to ##|\chi\rangle##. We'll be working with the two ordered bases ##B=\big(|+\rangle,|-\rangle\big)## and ##B'=\big(|\chi\rangle,|\phi\rangle\big)##.

It's straightforward to calculate ##[\rho]_B##. That's one of the things the problem asks us to do. I see that you've already done that, so I'll write down the result
$$[\rho]_B=\begin{pmatrix}|a|^2 & ab^*\\ a^*b & |b|^2\end{pmatrix}.$$

Regarding the rest of the problem: We can immediately tell that
$$[\rho]_{B'}=\begin{pmatrix}1 & 0\\ 0 & 0\end{pmatrix}.$$ So why don't we just define the rotation operator by ##U|+\rangle=|\chi\rangle## and ##U|-\rangle=|\phi\rangle##, and then use this to determine ##_B## in terms of a and b?

This worked out nicely for me. The only thing that bugs me about this is that the problem statement seems to require you to use the rotation operator to diagonalize ##\rho##. In my approach, we use the assumptions that ##\rho## is diagonal in the B' basis and that the B' basis is obtained by applying U to the elements of B, to find U. So U is the last matrix we find of all the matrices we're interested in. When we find it, there's nothing left to do.
 
  • #44
Fredrik said:
I found a solution, sort of. But maybe you guys are already past this stage. I haven't been following the thread closely.

Let a,b be the unique complex numbers such that ##|\chi\rangle=a|+\rangle+b|-\rangle##. We have ##\rho=|\chi\rangle\langle\chi|##. Let ##|\phi\rangle## be any unit vector that's orthogonal to ##|\chi\rangle##. We'll be working with the two ordered bases ##B=\big(|+\rangle,|-\rangle\big)## and ##B'=\big(|\chi\rangle,|\phi\rangle\big)##.

It's straightforward to calculate ##[\rho]_B##. That's one of the things the problem asks us to do. I see that you've already done that, so I'll write down the result
$$[\rho]_B=\begin{pmatrix}|a|^2 & ab^*\\ a^*b & |b|^2\end{pmatrix}.$$

Regarding the rest of the problem: We can immediately tell that
$$[\rho]_{B'}=\begin{pmatrix}1 & 0\\ 0 & 0\end{pmatrix}.$$ So why don't we just define the rotation operator by ##U|+\rangle=|\chi\rangle## and ##U|-\rangle=|\phi\rangle##, and then use this to determine ##_B## in terms of a and b?

This worked out nicely for me. The only thing that bugs me about this is that the problem statement seems to require you to use the rotation operator to diagonalize ##\rho##. In my approach, we use the assumptions that ##\rho## is diagonal in the B' basis and that the B' basis is obtained by applying U to the elements of B, to find U. So U is the last matrix we find of all the matrices we're interested in. When we find it, there's nothing left to do.


Yes...I was trying to guide fluid to this conclusion... I also assumed that what the problem meant by asking him to find the rotation matrix, is that he had to find the correspondence between the ##U## and a rotation in real space given by ##U=\cos(\theta/2)\hat{I}+\sin(\theta/2)\hat{n}\cdot\vec{S}##. So that's just one more step where after you find ##U## you have to decompose it in this way.
 
  • #45
I said earlier that every vector in this space is an eigenvector of ##\vec n\cdot\vec S##, where ##\vec n## is some unit vector in ##\mathbb R^3##. I just took a peek at some notes I made at least 5 years ago. I will give you some of the highlights, but no guarantee that the result is correct.

If we assume that ##|\chi\rangle =a|+\rangle+b|-\rangle## is a normalized eigenstate of ##\vec n\cdot\vec S##, with eigenvalue ##\lambda##, and that ##\vec n## is a unit vector, we can prove the following results:
\begin{align}
\lambda a&=\frac 1 2 (n^*b+n_3a)\\
\lambda b&=\frac 1 2 (na-n_3b)\\
\lambda &=\pm\frac 1 2.
\end{align} The ##n## in the first two equalities is defined by ##n=n_1+in_2##. My starting point was just to write down the eigenvalue equation with ##S_1## and ##S_2## expressed in terms of the raising and lowering operators ##S_\pm=S_1\pm iS_2##.

After deriving the results above, I verified that when those conditions are satisfied, ##|\chi\rangle## is an eigenstate with eigenvalue ##\frac 1 2## or ##-\frac 1 2##. So there are two eigenstates. I denoted them by ##|\vec n+\rangle## and ##|\vec n-\rangle##. The last thing I did was to find that
$$|\vec n\pm\rangle =N\left(|+\rangle+\frac{n_1+in_2}{n_3\pm 1}|-\rangle\right),$$ where N is a normalization constant that I didn't bother to determine.
 
  • #46
Fredrik said:
I said earlier that every vector in this space is an eigenvector of ##\vec n\cdot\vec S##, where ##\vec n## is some unit vector in ##\mathbb R^3##. I just took a peek at some notes I made at least 5 years ago. I will give you some of the highlights, but no guarantee that the result is correct.

If we assume that ##|\chi\rangle =a|+\rangle+b|-\rangle## is a normalized eigenstate of ##\vec n\cdot\vec S##, with eigenvalue ##\lambda##, and that ##\vec n## is a unit vector, we can prove the following results:
\begin{align}
\lambda a&=\frac 1 2 (n^*b+n_3a)\\
\lambda b&=\frac 1 2 (na-n_3b)\\
\lambda &=\pm\frac 1 2.
\end{align} The ##n## in the first two equalities is defined by ##n=n_1+in_2##. My starting point was just to write down the eigenvalue equation with ##S_1## and ##S_2## expressed in terms of the raising and lowering operators ##S_\pm=S_1\pm iS_2##.

After deriving the results above, I verified that when those conditions are satisfied, ##|\chi\rangle## is an eigenstate with eigenvalue ##\frac 1 2## or ##-\frac 1 2##. So there are two eigenstates. I denoted them by ##|\vec n+\rangle## and ##|\vec n-\rangle##. The last thing I did was to find that
$$|\vec n\pm\rangle =N\left(|+\rangle+\frac{n_1+in_2}{n_3\pm 1}|-\rangle\right),$$ where N is a normalization constant that I didn't bother to determine.

But I'm not sure how you go from this solution to finding a unitary rotation matrix ##U## since ##\vec{n}\cdot\vec{S}## is a Hermitian matrix, not a unitary one.

EDIT: After thinking for a minute, I see where you're going with this. You want to at the end rotate the ##z##-axis into ##\vec{n}## with some ##SO(3)## matrix to find "the rotation matrix"?
 
Last edited:
  • #47
I haven't actually worked that part of the problem, and I don't have a perfect understanding of rotation operators, but I'll offer some of my thoughts: If ##e^{i\theta S_z}## is a rotation by ##\theta## around the z axis (am I off by a factor of 2 or something?), then ##e^{i\vec\theta\cdot\vec S}## should be a rotation by ##|\vec\theta|## around the ##\vec\theta## axis. We can write ##\theta=|\vec\theta|##, ##\hat\theta=\vec\theta/\theta## and ##e^{i\vec\theta\cdot\vec S} =e^{i\theta(\hat\theta\cdot\vec S)}##.

So now it's a matter of determining ##\hat\theta## and then ##\theta##. We should be able to express ##\vec n## in terms of a,b. Then we can determine the plane that contains ##\vec 0##, ##\vec n## and ##\vec e_3##. The unit vector ##\hat\theta## should be orthogonal to that plane. ##\theta## is the angle between ##\vec n## and ##\vec e_3##.

This seems like a lot of work, and I doubt that the person who came up with the problem intended us to do something like this.
 
  • #48
Fredrik said:
I haven't actually worked that part of the problem, and I don't have a perfect understanding of rotation operators, but I'll offer some of my thoughts: If ##e^{i\theta S_z}## is a rotation by ##\theta## around the z axis (am I off by a factor of 2 or something?), then ##e^{i\vec\theta\cdot\vec S}## should be a rotation by ##|\vec\theta|## around the ##\vec\theta## axis. We can write ##\theta=|\vec\theta|##, ##\hat\theta=\vec\theta/\theta## and ##e^{i\vec\theta\cdot\vec S} =e^{i\theta(\hat\theta\cdot\vec S)}##.

Actually, you're not missing a factor of 2. The factor of 2 will come when you take ##\vec{S}=\frac{\hbar}{2}\vec{\sigma}##. The method in your post seems too complicated though.

I also noticed that I have been writing the wrong ##U## this whole time...the formula for ##U## in fluid's original post is what I meant...and nobody corrected me even!

...In fact I must apologize, since I have been guiding fluid into a longer way to solve this problem. I never bothered to solve this problem out myself, and now that I have, I found a much simpler way...

Fluid, start with the ##\hat{u}\cdot\vec{\sigma}## part of the equation, which you basically already got in post #19 (forget the factor of ##\hbar/2##). Now find ##U## (by that I mean construct ##U=\cos(\eta/2)\hat{I}-i\sin(\eta/2)\hat{u}\cdot\vec{\sigma}##). Now identify the left column of that final matrix with the column vector for ##\left|\chi\right>## and you're done. Because that left column will take ##\left|z_+\right>##, which in that basis is of course the column vector ##(1,0)##, to the most general spin state constructable, which for our problem is ##\left|\chi\right>##. I verified this since it is equation 14-23 of Baym's lectures on quantum mechanics.

We've been working way too hard to try to find the answer.
 
  • #49
Fredrik said:
Let a,b be the unique complex numbers such that ##|\chi\rangle=a|+\rangle+b|-\rangle##. We have ##\rho=|\chi\rangle\langle\chi|##. Let ##|\phi\rangle## be any unit vector that's orthogonal to ##|\chi\rangle##. We'll be working with the two ordered bases ##B=\big(|+\rangle,|-\rangle\big)## and ##B'=\big(|\chi\rangle,|\phi\rangle\big)##.

It's straightforward to calculate ##[\rho]_B##. That's one of the things the problem asks us to do. I see that you've already done that, so I'll write down the result
$$[\rho]_B=\begin{pmatrix}|a|^2 & ab^*\\ a^*b & |b|^2\end{pmatrix}.$$

Regarding the rest of the problem: We can immediately tell that
$$[\rho]_{B'}=\begin{pmatrix}1 & 0\\ 0 & 0\end{pmatrix}.$$ So why don't we just define the rotation operator by ##U|+\rangle=|\chi\rangle## and ##U|-\rangle=|\phi\rangle##, and then use this to determine ##_B## in terms of a and b?

A few more words about this approach, which I believe is the simplest one. The matrix equalities corresponding to ##U|+\rangle=|\chi\rangle## and ##U|-\rangle=|\phi\rangle## in the B basis are
\begin{align}&_B\begin{pmatrix}1\\ 0\end{pmatrix} =\begin{pmatrix}a \\ b\end{pmatrix}\\
&_B\begin{pmatrix}0\\ 1\end{pmatrix} =\text{ something that's orthogonal to }\begin{pmatrix}a \\ b\end{pmatrix}.
\end{align} This allows us to almost immediately write down ##_B## in terms of a,b. There's no need to figure out what the rotation axis is.

We can use the formula ##\big([\rho]_B\big)_{ij}=\langle e_i,\rho e_j\rangle## and the corresponding one for B' to prove that ##[\rho]_{B'}=[U^*]_B[\rho]_B _B##. We already know that all of these matrices look like, so we won't learn anything new if we perform the matrix multiplication on the right, unless we've made mistakes along the way.
 
  • #50
Fredrik said:
A few more words about this approach, which I believe is the simplest one. The matrix equalities corresponding to ##U|+\rangle=|\chi\rangle## and ##U|-\rangle=|\phi\rangle## in the B basis are
\begin{align}&_B\begin{pmatrix}1\\ 0\end{pmatrix} =\begin{pmatrix}a \\ b\end{pmatrix}\\
&_B\begin{pmatrix}0\\ 1\end{pmatrix} =\text{ something that's orthogonal to }\begin{pmatrix}a \\ b\end{pmatrix}.
\end{align} This allows us to almost immediately write down ##_B## in terms of a,b. There's no need to figure out what the rotation axis is.

We can use the formula ##\big([\rho]_B\big)_{ij}=\langle e_i,\rho e_j\rangle## and the corresponding one for B' to prove that ##[\rho]_{B'}=[U^*]_B[\rho]_B _B##. We already know that all of these matrices look like, so we won't learn anything new if we perform the matrix multiplication on the right, unless we've made mistakes along the way.


But solving it this way just gives you an arbitrary unitary matrix, you get the same exact matrix as how any arbitrary special unitary matrix can be written...which I posted in post #33. I don't see how this actually gives anything new to the problem, so I assumed that the problem was asking how does this unitary matrix correspond to a rotation in 3-space. o.o

Indeed this seems like a weird problem to me since ##\left|\chi\right>## is arbitrary.
 
  • #51
It's arbitrary only in the sense that ##|\chi\rangle## is arbitrary. The components are completely determined by ##|\chi\rangle##. If we're asked to find the rotation operator that diagonalizes ##\rho##, then this is the answer.

I think this is a plausible interpretation of the problem: Find a unitary U such that ##U|+\rangle=|\chi\rangle##. Then find ##_B## and verify that ##[U^*]_B[\rho]_B _B## is diagonal.

That second part can be done by explicitly doing the matrix multiplication, or by proving the formula ##[\rho]_{B'}=[U^*]_B[\rho]_B _B## for arbitrary bases B and B'.
 
  • #52
Fredrik said:
It's arbitrary only in the sense that ##|\chi\rangle## is arbitrary. The components are completely determined by ##|\chi\rangle##. If we're asked to find the rotation operator that diagonalizes ##\rho##, then this is the answer.

I think this is a plausible interpretation of the problem: Find a unitary U such that ##U|+\rangle=|\chi\rangle##. Then find ##_B## and verify that ##[U^*]_B[\rho]_B _B## is diagonal.

That second part can be done by explicitly doing the matrix multiplication, or by proving the formula ##[\rho]_{B'}=[U^*]_B[\rho]_B _B## for arbitrary bases B and B'.


Well if that's the case, then isn't the answer just the matrix in post 33?
 
  • #53
Matterwave said:
Maybe this information can help a little. Any arbitrary unitary matrix (in suggestive notation!) can be expressed as:

$$U=\begin{bmatrix} \alpha && -\bar{\beta} \\ \beta && \bar{\alpha} \end{bmatrix}$$

With the special unitary matrices satisfying the additional condition that the determinant is 1, or ##|\alpha|^2+|\beta|^2=1##.

Notice that this matrix rotates ##\left|z_+\right>## into ##\left|\chi\right>## already... and the left column is orthogonal to the right column...

Speaking of which, yes this essentially the solution to the problem. Note that what you are giving is already a special unitary matrix since its determinant is real and positive. The condition ##|\alpha|^2+|\beta|^2=1## is the condition that it is unitary at all. For an arbitrary unitary matrix, you need to multiply by an overall phase. In addition, one other phase may also be removed.
 
  • #54
Orodruin said:
Speaking of which, yes this essentially the solution to the problem. Note that what you are giving is already a special unitary matrix since its determinant is real and positive. The condition ##|\alpha|^2+|\beta|^2=1## is the condition that it is unitary at all. For an arbitrary unitary matrix, you need to multiply by an overall phase. In addition, one other phase may also be removed.

But if ##\alpha,\beta## are just 2 arbitrary complex numbers, it seems to me then that matrix should have 4 degrees of freedom, but SU(2) is of dimension 3. Where was 1 degree of freedom removed? I do see your point, but I'm not seeing where one degree of freedom was removed in writing the matrix in that form.
 
  • #55
Matterwave said:
But if ##\alpha,\beta## are just 2 arbitrary complex numbers, it seems to me then that matrix should have 4 degrees of freedom, but SU(2) is of dimension 3. Where was 1 degree of freedom removed? I do see your point, but I'm not seeing where one degree of freedom was removed in writing the matrix in that form.

##\alpha## and ##\beta## are not completely arbitrary if the matrix is to be unitary. For the matrix to be unitary at all, you have the condition on their square modulus, which reduces the degrees of freedom to 3. The point being that the 4-degrees-of-freedom matrix is not unitary without this condition. However, if the matrix is unitary, then it is also special unitary and you need to add the overall phase to get a general unitary matrix.
 
  • #56
Matterwave said:
Well if that's the case, then isn't the answer just the matrix in post 33?
Yes, but you also need to mention that the ##\alpha## and ##\beta## in that matrix are the components of ##|\chi\rangle## in the B basis. ##|\chi\rangle=\alpha|+\rangle+\beta|-\rangle##.

I used the notation ##|\chi\rangle=a|+\rangle+b|-\rangle##, so my a,b are your ##\alpha,\beta##. I didn't just write down the most general form of a unitary matrix. I used the following argument:

Fredrik said:
The matrix equalities corresponding to ##U|+\rangle=|\chi\rangle## and ##U|-\rangle=|\phi\rangle## in the B basis are
\begin{align}&_B\begin{pmatrix}1\\ 0\end{pmatrix} =\begin{pmatrix}a \\ b\end{pmatrix}\\
&_B\begin{pmatrix}0\\ 1\end{pmatrix} =\text{ something that's orthogonal to }\begin{pmatrix}a \\ b\end{pmatrix}.
\end{align}

This gave me the result
$$_B=\begin{pmatrix}a & b^*\\ b & -a^*\end{pmatrix}.$$ You put the minus sign on the upper right instead of the lower right. That works just as well. The quoted argument says only that we need to choose the second column of ##_B## so that it's orthogonal to the first. Since (b*,-a*) and (-b*,a*) are both orthogonal to (a,b), we can take either one as the second column of ##_B##.

I should also have mentioned that the second column must be normalized (because unitary operators take elements of an orthonormal set (like {(1,0),(0,1)}) to elements of an orthonormal set). We can use any ##e^{ir}(b^*,-a^*)## with ##r\in\mathbb R## as the second column. My choice corresponds to r=1, yours to r=##\pi##.

Edit: As I was writing my next post below, I realized that the determinant of ##_B## is determined by the "arbitrary" parameter r. We have ##\det _B=e^{-ir}(-|a|^2-|b|^2)##. My choice of r makes this equal to -1. Your choice makes it equal to 1. So if we just want to diagonalize ##\rho##, either choice is fine, but if we want to do it with a rotation operator (as the problem says), we have to go with your choice. (Because rotation operators are by definition elements of SU(2)). So I chose the wrong sign for the second column of my ##_B##. This is the correct ##_B##:
$$_B=\begin{pmatrix}a & -b^*\\ b & a^*\end{pmatrix}.$$
Matterwave said:
But if ##\alpha,\beta## are just 2 arbitrary complex numbers, it seems to me then that matrix should have 4 degrees of freedom, but SU(2) is of dimension 3. Where was 1 degree of freedom removed? I do see your point, but I'm not seeing where one degree of freedom was removed in writing the matrix in that form.
They're not arbitrary. They're defined by ##|\chi\rangle=\alpha|+\rangle+\beta|-\rangle##. (OK, so they're arbitrary in the sense that ##|\chi\rangle## is arbitrary, but they're completely determined by ##|\chi\rangle##). 1 degree of freedom is removed by the requirement that ##|\chi\rangle## is normalized.
 
Last edited:
  • #57
Perhaps it's worth pointing out that that a general density operator for this situation takes the form
[tex]\rho = \frac{1}{2}({\bf 1} + {\bf a} \cdot {\bf \sigma})[/tex]
where [itex]{\bf a}[/itex] is a unit vector on the Bloch sphere.

In this language, the problem reduces to simply finding the SO(3) rotation matrix that takes the 3-vector [itex]{\bf a}[/itex] into the z-axis.
 
  • #58
btw fluidistic - this online lecture should tell you everything you need to know about density operators, traces, Bloch sphere etc etc...

http://pirsa.org/10090019/
 
  • #59
Fredrik said:
They're not arbitrary. They're defined by ##|\chi\rangle=\alpha|+\rangle+\beta|-\rangle##. (OK, so they're arbitrary in the sense that ##|\chi\rangle## is arbitrary, but they're completely determined by ##|\chi\rangle##). 1 degree of freedom is removed by the requirement that ##|\chi\rangle## is normalized.

In that particular part of this discussion, ##\alpha## and ##\beta## were not related to any state but used to parametrize a general special unitary matrix so the argument should not be based on the state ##|\chi\rangle##. The question was why the matrix
$$
\begin{pmatrix} \alpha & - \bar \beta \\ \beta & \bar\alpha\end{pmatrix}
$$
with 4 degrees of freedom could only be a parametrization of the special unitary group (3 dof) and not a general unitary matrix (4 dof). The answer to this was given in my last post: Being unitary at all requires that the modulus of the determinant is one:
$$
\left| |\alpha|^2 + |\beta|^2\right| = |\alpha|^2 + |\beta|^2 = 1,
$$
which happens to be the same as the requirement of the special unitary group, i.e., if we call the set of matrices of the above form ##A##, then ##A \cap U(2) = SU(2)##.
 
  • #60
Orodruin said:
In that particular part of this discussion, ##\alpha## and ##\beta## were not related to any state but used to parametrize a general special unitary matrix so the argument should not be based on the state ##|\chi\rangle##.
He asked if what I had said meant that simply writing down the general form of a unitary matrix solves the problem. I'm assuming that this was meant to be an argument against my approach, since it clearly doesn't solve the problem. So I explained how and why my approach solves the problem.

Orodruin said:
The question was why the matrix
$$
\begin{pmatrix} \alpha & - \bar \beta \\ \beta & \bar\alpha\end{pmatrix}
$$
with 4 degrees of freedom could only be a parametrization of the special unitary group (3 dof) and not a general unitary matrix (4 dof). The answer to this was given in my last post: Being unitary at all requires that the modulus of the determinant is one:
$$
\left| |\alpha|^2 + |\beta|^2\right| = |\alpha|^2 + |\beta|^2 = 1,
$$
which happens to be the same as the requirement of the special unitary group, i.e., if we call the set of matrices of the above form ##A##, then ##A \cap U(2) = SU(2)##.
OK, I see that that part of my reply to Matterwave looks weird, since I quoted the same question and gave him a different answer. Your answer is appropriate in a discussion about Matterwave's approach. I gave him the answer that's appropriate in a discussion about my approach.

Thinking about what you said also made me see that there's a detail in my previous post that I need to elaborate on. I have edited in a comment about it now.
 
Last edited:
  • #61
Orodruin said:
##\alpha## and ##\beta## are not completely arbitrary if the matrix is to be unitary. For the matrix to be unitary at all, you have the condition on their square modulus, which reduces the degrees of freedom to 3. The point being that the 4-degrees-of-freedom matrix is not unitary without this condition. However, if the matrix is unitary, then it is also special unitary and you need to add the overall phase to get a general unitary matrix.

I see what you mean. So to clarify for my own sake, the matrix I wrote down can ONLY be used to specify a SU(2) matrix, and not a U(2) one because it is either SU(2) (given ##|\alpha|^2+|\beta|^2=1##) or not unitary at all (if the previous condition is violated), am I right? A general U(2) matrix has the form of the matrix I wrote down, along with the requirement ##|\alpha|^2+|\beta|^2=1##, AND multiplied by an overall phase ##e^{i\varphi}## correct?
 
  • #62
Fredrik said:
He asked if what I had said meant that simply writing down the general form of a unitary matrix solves the problem. I'm assuming that this was meant to be an argument against my approach, since it clearly doesn't solve the problem. So I explained how and why my approach solves the problem.

I was not really arguing "against" your approach, so to speak. In fact, your approach was the way I was trying to lead fluid in the first place (and also why I wrote in my post #33 such a matrix in the first place, and remarked that the first column was basically ##\left|\chi\right>## and that it was orthogonal to the second column). I was hoping that fluid would realize that such a matrix solves his problem (I could not figure out a way to guide him to that matrix...).

But what I did mean was that such an approach might not be enough for what the question was asking for, since it asked for a "rotation" (otherwise, I would just be giving fluid the answer to his homework problem).

Of course, a "rotation" on 2-component spinor space IS just some matrix in SU(2). But, I thought the question also wanted you to correspond such a rotation with a rotation in real space. I.e. I assumed the question also wanted you to make a correspondence with a matrix (find a matrix) in SO(3) as well.

So I would do everything with your approach, and then construct also the SU(2) matrix ##U=\cos(\eta/2)\hat{I}-i\sin(\eta/2)\hat{u}\cdot\vec{\sigma}## and make a correspondence between the two matrices.
 
Last edited:
  • #63
Matterwave said:
Of course, a "rotation" on 2-component spinor space IS just some matrix in SU(2).
Right, so maybe the word "rotation" in the problem statement is just telling us to choose that ##r## in post #56 so that the matrix gets the determinant 1.

Matterwave said:
But, I thought the question also wanted you to correspond such a rotation with a rotation in real space. I.e. I assumed the question also wanted you to make a correspondence with a matrix (find a matrix) in SO(3) as well.

So I would do everything with your approach, and then construct also the SU(2) matrix ##U=\cos(\eta/2)\hat{I}-i\sin(\eta/2)\hat{u}\cdot\vec{\sigma}## and make a correspondence between the two matrices.
Have you found a way to do that (other than the complicated method that I described but didn't carry out)? I haven't, but I haven't really tried yet.
 
  • #64
Fredrik said:
Right, so maybe the word "rotation" in the problem statement is just telling us to choose that ##r## in post #56 so that the matrix gets the determinant 1.Have you found a way to do that (other than the complicated method that I described but didn't carry out)? I haven't, but I haven't really tried yet.

I mean at the end of that calculation you have a matrix exactly of the form in post #33, so you can make an easy identification with ##\alpha,\beta##. Frist we want to construct ##U=\cos(\eta/2)\hat{I}-i\sin(\eta/2)\hat{u}\cdot\vec{\sigma}##. We can use spherical coordinates for ##\hat{u}## so that it has components ##\hat{u}=(\sin\theta\cos\phi,\sin\theta\sin\phi,\cos\theta)## where ##\theta## is the azimuthal angle, and ##\phi## is the polar angle.

Thus, assuming I did all the calculations right, the final matrix you get at the end is:

$$U=\begin{pmatrix} \cos(\eta/2)-i\cos(\theta)sin(\eta/2) & -i\sin(\eta/2)\sin(\theta)e^{-i\phi} \\ -i\sin(\eta/2)\sin(\theta)e^{i\phi} & \cos(\eta/2)+i\cos(\theta)\sin(\eta/2)\end{pmatrix}$$

Therefore, we simply make the identifications:

$$\alpha=\cos(\eta/2)-i\cos(\theta)sin(\eta/2),~~\beta= -i\sin(\eta/2)\sin(\theta)e^{-i\phi}$$

So that now we know how a real space rotation given by a rotation of angle ##\eta## around an axis of rotation given by ##\theta,\phi## corresponds to a rotation in spin space from an eigenstate ##\left|z_+\right>## to ##\left|\chi\right>##, some arbitrary spin state.
 

Similar threads

  • Advanced Physics Homework Help
Replies
7
Views
1K
Replies
9
Views
2K
  • Advanced Physics Homework Help
Replies
4
Views
1K
  • Quantum Physics
Replies
4
Views
1K
  • Advanced Physics Homework Help
Replies
9
Views
913
  • Calculus and Beyond Homework Help
Replies
16
Views
1K
Replies
2
Views
484
  • Advanced Physics Homework Help
Replies
1
Views
2K
  • Advanced Physics Homework Help
Replies
1
Views
1K
Replies
1
Views
1K
Back
Top